A corporation distributes a 10% common stock dividend on 30000 shares issued when the market value of its common stock is $24 per share and its par value is $2 per share dollars per share on the distribution date a credit for $___ would be journalized.A. $30,000B. $6,000C. $72,000D. $66,000

Answers

Answer 1

A corporation distributes a 10% common stock dividend on 30,000 shares.

The market value is $24 per share.

The par value is $2 per share.

We have to find the credit that is journalized the moment the distribution is made.

They paid a total amount in dividends that is 10% of the par value of the stock times the number of stocks:

[tex]\begin{gathered} 10\%\cdot2\cdot30000 \\ 0.1\cdot2\cdot30000 \\ 6000 \end{gathered}[/tex]

Answer: the credit is $6,000 [Option B]


Related Questions

One equation from a system of two linear equations is graphed on the coordinate grid. 51 46 5 4 3 2 1 6 x -1 -21 The second equation in the system of linear equations has a slope of 3 and passes through the point (2,-5). What is the solution to the system of equations? th

Answers

First, we need to find the equation for the two equations.

The equation graphed has a y-intercept of 3 and a slope of

[tex]m=\frac{-6}{3}=-3[/tex]

therefore, the equation of the line is

[tex]\boxed{y=-\frac{1}{2}x+3.}[/tex]

For the second equation, we know what it has a slope of 3; therefore it can be written as

[tex]y=3x+b[/tex]

Now, we also know that this equation passes through the point y = -5, x = 2; therefore,

[tex]-5=3(2)+b[/tex]

which gives

[tex]-5=6+b[/tex][tex]b=-11[/tex]

Hence, the equation of the line is

[tex]\boxed{y=3x-11}[/tex]

Now we have the equations

[tex]\begin{gathered} y=-\frac{1}{2}x+3 \\ y=3x-11 \end{gathered}[/tex]

equating them gives

[tex]-\frac{1}{2}x+3=3x-11[/tex]

adding 11 to both sides gives

[tex]-\frac{1}{2}x+14=3x[/tex]

adding 1/2 x to both sides gives

[tex]14=\frac{7}{2}x[/tex]

Finally, dividing both sides by 7/2 gives

[tex]\boxed{x=4\text{.}}[/tex]

The corresponding value of y is found by substituting the above value into one of the equations

[tex]y=-\frac{1}{2}(4)+3[/tex][tex]y=1[/tex]

Hence, the solution to the system is

[tex](4,1)_{}[/tex]

Suppose f(x) = x². Find the graph off(x+3).???

Answers

If f(x)=x^2

Then f(x+3)=(x+3)^2

[tex](x+3)^2=x^2+6x+9[/tex]

Use geogebra to graph the function or calculate the vertex using the equation

x=-b/2a

From the equation we have that

b=6

a=1

x=-6/(2*1)

x=-3

The vertex is on x=-3

Calculate f(-3)=9-18+9=0

The vertex is (-3,0)

y axis cut off point f(0)=0+0+9=9

As "a" is a positive value, parabola open upwards, now you can draw the parabola

This is a sketch, let's use geogebra

Three points are shown on the coordinate plane.What is the distance from point A to point B?

Answers

Answer:

The distance from point A to point B is;

[tex]5\text{ units}[/tex]

Explanation:

Given the points A and B with coordinates as shown on the graph;

[tex]\begin{gathered} A(0,5) \\ B(3,1) \end{gathered}[/tex]

Recall that the distance between two points can be calculated using the formula;

[tex]d=\sqrt[]{(x_2-x_1)^2+(y_2-y_1)^2}[/tex]

substituting the coordinates of point A and B. we have;

[tex]\begin{gathered} d=\sqrt[]{(3-0)^2+(1-5)^2} \\ d=\sqrt[]{3^2+4^2} \\ d=\sqrt[]{9+16} \\ d=\sqrt[]{25} \\ d=5 \end{gathered}[/tex]

Therefore, the distance from point A to point B is;

[tex]5\text{ units}[/tex]

If x is perpendicular to a and X is perpendicular to b then____X is perpendicular to a A // BA is perpendicular to YX // Y

Answers

[tex]If\text{ x}\perp a\text{ and x}\perp b,\text{ then a}\parallel b[/tex]

There are two buildings beside a park.
The first building is 165 3/4 ft tall, and
the second building is 114 1/4 ft tall.
By rounding to the nearest whole number,
estimate the difference between the
heights of the buildings.

Answers

The difference of the height of the buildings is 51.5 for the first building is 165 3/4 feet tall, and the second building is 114 1/4 feet tall.

Given that,

There are two building side of a park.

The first building is 165 3/4 feet tall, and the second building is 114 1/4 feet tall.

We have to find the difference of the height of the buildings.

We have to subtract the first building is 165 3/4 feet tall, and the second building is 114 1/4 feet tall.

165 3/4- 114 1/4

660+3/4- 456+1/4

663/4-457/4

165.75-114.25

51.5

Therefore, The difference of the height of the buildings is 51.5 for the first building is 165 3/4 feet tall, and the second building is 114 1/4 feet tall.

To learn more about height visit: https://brainly.com/question/10726356

#SPJ9

what is the quotient of the complex numbers below 3 + 2i / 1 - 5i

Answers

[tex]\frac{3+2i}{1-5i}[/tex]

Take the conjugate of the denominator, use it to multiply the numerator and the denominator

That is;

[tex]undefined[/tex]

Answer:

Step-by-step explanation:

Lakshmi bought 7 books for a total of 56 rupees how much would see pay for just three books? 56 rupees Indian money

Answers

To find how much would be paid for 3 books, follow the steps below.

Step 01: Find the price of one book.

Let's say the price of one book is x.

Then, the price of 7 books is 7 times x, which is 56 rupes.

[tex]7x=56[/tex]

To find x, let's divide both sides by 7:

[tex]\begin{gathered} \frac{7x}{7}=\frac{56}{7} \\ 1x=8 \\ x=8 \end{gathered}[/tex]

So, the price of one book is 8.

Step 02: Find the price of 3 books.

If the price of one book is 8, the price of 3 books (P) will be 3 times 8:

[tex]\begin{gathered} P=3\cdot8 \\ P=24 \end{gathered}[/tex]

Answer: It would be paid 24 rupees for 3 books.

what is x^3 - 2x^2 - 4x - 1 divided by x + 1 ?

Answers

To answer this question we will use the long division.

Using long division we get:

Therefore:

[tex]\frac{x^3-2x^2-4x-1}{x+1}=x^2-3x-1.[/tex]

Answer: Option A.

jenelle and hadiya went to lunch,the bill,before sales before sales tax and tip,was 37.50.a sales tax of 8% was added.they added an 18% tip on the amount after the tax was added.a)what was the amount,in dollars,of the sales tax.b)what was the total amount they paid,including tax and tip.

Answers

$37.50

tax = 8%

tip = 18%

a) 37.5 ------------------ 100%

x ------------------- 8%

x = (8 x 37.5) / 100

x = 3

Tax = $3

b) Money of lunch plus tax = $40.5

40.5 -------------------- 100

x --------------------- 18

x = (18 x 40.5) / 100

x = 7.29

Total amount paid = 7.29 + 40.5

= $ 47.79

Hello, I need some assistance with this homework question please for precalculusHW Q15

Answers

Solution

The remainder theorems state that when a polynomial a(x) is divided by a linear polynomial b(x) whose zero is x = k, the remainder is given by r = a(k).

Given

[tex]f(x)=4x^3-10x^2+10x-4[/tex]

since f(x) is divided by x - 2, the remainder is

[tex]f(2)=4(2)^3-10(2)^2+10(2)-4=4(8)-10(4)+20-4=32-40+20-4=8[/tex]

Therefore, the remainder is 8

Find the cost for each pound of jelly beans and each pound of almonds

Answers

Let 'x' represent the cost for each pound of jelly beans.

Let 'y' represent the cost for each pound of almonds.

For the first statement, the mathematical expression is

[tex]\begin{gathered} 9x+7y=37\ldots\ldots1 \\ \end{gathered}[/tex]

For the second statement, the mathematical expression is,

[tex]3x+5y=17\ldots\ldots2[/tex]

Combining the two equations

[tex]\begin{gathered} 9x+7y=37\ldots\ldots\text{.}.1 \\ 3x+5y=17\ldots\ldots2 \end{gathered}[/tex]

Applying the elimination method to resolve the systems of equation

Multiply the second equation by 3, in order to eliminate x

[tex]\begin{gathered} 9x+7y=37\ldots\ldots\ldots1 \\ 3x+5y=17\ldots\ldots\ldots2\times3 \end{gathered}[/tex][tex]\begin{gathered} 9x+7y=37\ldots\ldots\text{.}.1 \\ 9x+15y=51\ldots\ldots2 \end{gathered}[/tex]

Subtract equation 1 from 2

[tex]\begin{gathered} 9x-9x+15y-7y=51-37 \\ 8y=14 \\ \frac{8y}{8}=\frac{14}{8} \\ y=\frac{7}{4}=1.75 \\ \therefore y=1.75 \end{gathered}[/tex]

Substitute y = 1.75 into equation 1 in order to solve for x

[tex]\begin{gathered} 9x+7y=37 \\ 9x+7(1.75)=37 \\ 9x+12.25=37 \\ 9x=37-12.25 \\ 9x=24.75 \\ \frac{9x}{9}=\frac{24.75}{9} \\ x=2.75 \end{gathered}[/tex]

Hence, the cost for each pound of jelly beans = x = $2.75.

the cost for each pound of almonds = y = $1.75.

Write a coordinate proof of the following theorem:"If a quadrilateral is a kite, then its diagonals are perpendicular."(image attached)thank you ! :)

Answers

Explanations:

Given the following coordinate points from the kite

[tex]\begin{gathered} W=(a,4b) \\ X=(2a,b) \\ Y=(a,0) \\ Z=(0,b) \end{gathered}[/tex]

For the diagonals to be perpendicular the product of the distance WY and XZ must be zero that is;

[tex]\vec{WY}\cdot\vec{XZ}=0[/tex]

Determine the coordinate point WY

[tex]\begin{gathered} \vec{WY}=[(a-a,4b-0)] \\ \vec{WY}=(0,4b) \end{gathered}[/tex]

Determine the coordinate point XZ

[tex]\begin{gathered} \vec{XZ}=[(2a-0),(b-b)] \\ \vec{XZ}=(2a,0) \end{gathered}[/tex]

Take the dot product of the coordinates

[tex]\begin{gathered} \vec{WY}\cdot\vec{XZ}=(0,4b)\cdot(2a,0) \\ \vec{WY}\cdot\vec{XZ}=[(0)(2a)+(4b)(0))] \\ \vec{WY}\cdot\vec{XZ}=(0,0)=\vec{0} \end{gathered}[/tex]

Since the dot product of the coordinates is a zero vector, hence its diagonals are perpendicular.

Find the length of side x in simplest radical form with a rational denominator.30°х60°12

Answers

Ok, to find the lenght of side x we are going to use the sine function:

[tex]\sin (30)=\frac{12}{x}[/tex]

Clearing x:

[tex]x=\frac{12}{\sin (30)}=\frac{12}{1/2}=24[/tex]

Finally we get that x is equal to 24.

the point M 6,-4 is reflected over the y-axis. what are the cordnates of the resulting point, M?

Answers

A reflection over the Y-axis is given by:

[tex](x,y)\rightarrow(-x,y)[/tex]

Substitute for (x,y)=(6,-4):

[tex](6,-4)\rightarrow(-6,-4)[/tex]

Therefore, the new coordinates of the point after a reflection over the Y-axis, are:

[tex](-6,-4)[/tex]

Can u please help me with This am trying to study but can’t get it

Answers

Given:

Following Matrices are given.

[tex]A=\begin{bmatrix}{2} & {1} \\ {3} & {4}\end{bmatrix},B=[\text{ }5\text{ 4 \rbrack, C=}\begin{bmatrix}{4} & {1} & {6} \\ {} & {} & {} \\ {5} & {2} & {7}\end{bmatrix}[/tex]

Find:

we have to find which Matrix multiplication can be defined.

Explanation:

For Matrix multiplication, the number of columns of first Matrix should be equal to the number of rows of the second Matrix.

Therefore, the following Matrix multiplication can be defined

BC,Because number of columns of B is 2 and number of rows of C is 2.

AC,Because number of columns of A is 2 and number of rows of C is 2.

BA,Because number of columns of B is 2 and number of rows of A is 2.

Therefore, the multiplications BC,AC,BA can be defined.

Let c(t) be the number of customers in a restaurant t hours after 8 a.m. Explain the meaning of each statement.c(3)=c(3)

Answers

Given:

Here, c(t) be the number of customers in a restaurant t hours after 8 a.m.

The statement is,

[tex]c\left(3\right)=c\left(3\right)[/tex]

To find:

The meaning of the given statement.

Explanation:

Since c(t) is the number of customers in a restaurant t hours after 8 a.m.

So, c(3) be the number of customers in a restaurant 3 hours after 8 a.m.

That is,

The number of customers in a restaurant 3 hours after 8 a.m is equal to the number of customers in a restaurant 3 hours after 8 a.m.

Final answer:

The number of customers in a restaurant 3 hours after 8 a.m is equal to the number of customers in a restaurant 3 hours after 8 a.m.

The length of a rectangle is 1m more than twice the width, the area of the rectangle is 45m^2

Answers

Let l and w be the length and width of the rectangle, respectively; therefore, according to the question

[tex]\begin{gathered} l=2w+1 \\ and \\ l*w=45 \end{gathered}[/tex]

Where l and w are in meters.

Substitute the first equation into the second one, as shown below

[tex]\begin{gathered} l=2w+1 \\ \Rightarrow(2w+1)*w=45 \\ \Rightarrow2w^2+w=45 \\ \Rightarrow2w^2+w-45=0 \end{gathered}[/tex]

Solve for w using the quadratic formula,

[tex]\begin{gathered} \Rightarrow w=\frac{-1\pm\sqrt{1+4*2*45}}{2*2}=\frac{-1\pm\sqrt{361}}{4}=\frac{-1\pm19}{4} \\ \Rightarrow w=\frac{9}{2},-5 \end{gathered}[/tex]

But w has to be positive since it is a measurement; therefore, w=9/2.

Finding l given the value of w=9/2,

[tex]\begin{gathered} w=\frac{9}{2} \\ \Rightarrow l=2(\frac{9}{2})+1=10 \\ \Rightarrow l=10 \end{gathered}[/tex]

Thus, the answers are length=10 m, width=4.5m

Ravi had 119 dollars to begin with. He just spent b dollars.using. B, write expression for the number of dollars he has left

Answers

Given:

Total money Ravi has to begin with = 119 dollars.

He spent b dollars.

The number of dollars he has left is:

119 - b

If f(x) = -x² - 2x, what is f(-2)?

Answers

Answer: 0

Step-by-step explanation:

f(-2)= -(-2)^2-2(-2)

= -4+4=0

Solve for 3x/2 -4 = 16what does x equal??

Answers

The given equation is expressed as

3x/2 - 4 = 16

The first step is to multiply both sides of the equation by 2. It becomes

3x/2 * 2 - 4 * 2 = 16 * 2

3x - 8 = 32

3x = 32 + 8

3x = 40

x = 40/3

x = 13.33

Which situation is best modeled by the graph? a.) the cost of buying muffinsb.) the distance between the train and the station as the train travels towards the stationc.) the amount of money left in the roll of quarters after paying a roll each dayd.) the distance a runner covers traveling at a steady paste

Answers

Notice that the graph plots points that as we move along the horizontal axis, go down in value. The Horizontal axis is most likely representing the time elapsed in each description.

Then, we DISCARD the first answer, since the cost of muffins don't go down as time goes by.

Answer b is a POSSIBLE answer, since the distance as the train approaches the station, is reducing (going down in value) as time goes by.

Answer C is not a good answer (we discard it) since after paying a roll each day, the number of quarters in each roll doesn't go down because we pay with the entire roll.

Answer d is also discarded, since the distance covered by the runner, should be going UP (increasing) as time goes by

Therefore, answer b) is the selected answer.

does this represent exponential growth or exponential decay and identify the percent rate of changedetermine whether y= 500(1.08)t represents exponential growth or exponential decay and identify the rate of change.

Answers

Given:

[tex]500\mleft(1.08\mright)^t[/tex]

To determine whether it represents exponential growth or exponential decay:

Since, the general exponential growth formula is,

[tex]f\mleft(x\mright)=a\mleft(1+r\mright)^x[/tex]

Hence, the given represents exponential growth.

Comparing we get,

1+r=1.08

r=0.08

That is, r=8%

Therefore, the percentage rate of change is 8%.

3. Which expression is equivalent to 3(x-2) + Zx?A. -XB. 3xC. 5X-2D. 5X-6marios The cost of

Answers

Given an expression below :

[tex]3(x-2)+2x[/tex]

The expression can be solved by :

Step 1: Opening the bracket

[tex]\begin{gathered} 3(x-2)+2x \\ 3x-6+2x \end{gathered}[/tex]

Step 2: Collect like terms

[tex]\begin{gathered} 3x-6+2x \\ 3x+2x-6 \\ 5x-6 \end{gathered}[/tex]

Therefore the correct answer for the expression is 5x - 6

Hence the correct value is Option D

The figure below was made with a scale of 1 unit = 9 cm.Draw the figure with a new scale of 1 unit = 3 cm.You can place your figure anywhere on the grid on the right.9 cmCurrent scale1 unit = 9 cmExplanationCheck3 cmNew scale1 unit = 3 cmI need help with this math problem

Answers

We will draw the figure

In a new scale, the new scale is

[tex]1\text{ unit= 3 cm}[/tex]

Note that the draw above is a square of side 18cm, therefore in the new scale the side of the square have to be drawn using

[tex]\frac{18cm}{3cm}=6\text{ }units[/tex]

That is, if we change the units our new square have a side of 6 units as follows

What type of model does the data suggest?х01234y2.55102040A ConstantB ExponentialCLinearD) Quadratic

Answers

[tex]\begin{gathered} x=0,1,2,3,4 \\ y=2.5,5,10,20 \end{gathered}[/tex]

The function for this data can be represented as:

[tex]y=5\times2^{n-2}[/tex]

Therefore, it is exponential.

Find the volume of thetriangular prism.24 m7 m3.6 mThe volume of the triangular prism ism3

Answers

The volume of a triangle prism formula is shown below.

[tex]\text{Volume of a triangular prism = Base area x Lenght}[/tex]

From the figure,

The triangle of base 3.6m and height 24m is the base of the prism.

Therefore, the base area is the area of the triangle.

Area of the triangle = 1/2 x base x height

Area = 1/2 x 24 x 3.6

= 12 x 3.6

= 43.2

The volume = Base area x Length

Length = 7m

Base area = 43.2 meter square

Therefore,

The violume = 43.2 x 7

= 302.4

Final amswer

[tex]\text{Volume = 302.4 m}^3[/tex]

Solve the equation 42+7c - 5 = 0 using the quadratic formula

Answers

The equation is:

[tex]4c^2+7c-5=0[/tex]

so we can use the cuadratic equation so:

[tex]\begin{gathered} c=\frac{-b\pm\sqrt[]{b^2-4(a)(c^{\prime})}}{2(a)} \\ \text{where} \\ a=4 \\ b=7 \\ c^{\prime}=-5 \end{gathered}[/tex]

So if we replace in the equation we will have:

[tex]c=\frac{-7\pm\sqrt[]{7^2-4(4)(-5)}}{2(4)}[/tex]

So we simplify to solve the problem so:

[tex]c=\frac{-7\pm\sqrt[]{129}}{8}[/tex]

Evan is going to the 50th state fair this weekend. It costs $10 to enterand each ride is $2. How much will it cost Evan to go to the fair and ride 5rides? **Don't forget the initial cost.**( hint: determine the equation first y =X + and then plug in 5 for x) *

Answers

From the question, we are given the following;

Cost of entering the state fair = $10

Amount of each ride = $2

For us to determine the amount it will cost Evan to go to the fair and ride 5 rides, the equation y = $10 + 2x will be used where;

x is the total ride taken = 5 rides

y is the amount it cost evan to enter the state fair and ride 5 rides

Substitute x = 5 into the equation and get y;

y = $10 + 2x

y = $10 + $2(5)

y = $10 + $10

y = $20

Hence it will cost Evans $20 to go to the fair and ride 5 rides

how do I multeply Fractions

Answers

Multiplying simultaneously numerator by numerator and denominator by denominator.

You can multiply fractions, multiplying simultaneously numerator by numerator and denominator by denominator.

2) Notice that whenever possible, we must simplify it to the lowest possible fraction.

Mr.Ortiz has to successfully interview 90% of his assigned households. He was assigned 500 households. He has interviewed 430 households so far. Has he met his goal?

Answers

90 % of 500 is found by

0.9 * 500 = 450

430 < 450

so he has not met his goal

Other Questions
Explain in writing how the graph of the function h(x) = -|x-1| +4 is related to the graph of oneof the six basic functions. Sketch a graph of h(x). (6 points What role do totipotent cells play in the development of an animal?A. They produce proteins for an organism's growth and survival.B. They can become any type of cell in the body or in the placenta orumbilical cord.C. They work together to perform distinct functions in the animal.D. They produce the three germ layers of an embryo. If A= { 1,2,4,5,7,9} and B= {2,3,4} and U = {1,2,3,4,5,6,7,8,9} Find A The flooded rivers of Mesopotamia carried small particles of rich soil called____.A. sandB. siltC. clayD. gravel What are the two capabilities of the internet that promote and sustain customer relationships?. Which of the following are examples of qualitative observations? Select all that apply.The temperature is 95.2 C.The solution is green.The liquid is odorless.The volume is 5.6 L The percent y (in decimal form) of battery power remaining x hours after you turn on a laptop computer is y=-0.2x + 1. Graph the equation and use it to answer questions 1 and 2. a. What is the x-intercept? What does it represent? b. What is the y-intercept? What does it represent?c. After how many hours is the battery power at 75%d. what is the percentage of battery power remaining at 3 hours? which question is most closely related to the field of biology?A.) what is the nearest planet to earth?B.) how do you design a cheaper automobile?C.) how many planets are there in the solar system?D.) what is the cause of cancer in mice? y=8x+3 ordered pairs Derive the following functionsa) (2x+1)b) f(x) = [tex] \sqrt{6x + 3} [/tex] suppose that R(x) is a polynomial of degree 12 whose coefficients are real numbers. Also, suppose that R(x) has the following zeros. Answer the following.Edit: if its ok please double-check the answers. COMPARING METHODS Consider the equation x+ 2 = 3x 4.a. Solve the equation using algebra. Ab and CD are opposite sides so ab are congruent to CD 2a=34 a=17 find the value of each variable Because consumers often do not pay close attention to advertising messages, some of which contain large amounts of relatively complex information, advertisers are concerned withMultiple Choiceclutter.frequency.ratings.CPM.target markets. The following sets of data show the high temperatures in four different cities for a week in December Hello, I need some assistance with this homework question please for precalculusHW 23 why have u added mgsin thetha to calculate total force Is the following sentence in active voice or passivevoice? The ship sailed into the harbor carefully andsafely...*activepassive PepsiVinegarOrange JuiceWhat property do all three of these common household substances have in common? aThey all have a pH above 7. bThey all taste bitter. cThey are all acidic. dThey all turn litmus paper blue.Pls Helppp :( Read the passage from Rosa Parks.Shortly after 5:00 p.m., Rosa Parks clocked out of work and walked the block to Court Square to wait for her bus home. It had been a hard day, and her body ached, from her feet swollen from the constant standing to her shoulders throbbing from the strain and her chronic bursitis.What is a correct inference you can make based on this passage?A) Parks is leaving work early because of her health problems.B) Parks's hard day has put her in a bad mood, which causes her to defy the bus driver. C) Parks's job unfairly forces her to stay on her feet all day without breaks.D) Parks has a strong desire and need to sit, rather than stand, on the bus ride home.